You are interested in conducting a survey at your school to determine if gender or ethnicity is related to perceptions about educational opportunity. The school has 1,800 students but you only have resources to interview 100. The student body is approximately equally divided between males and females. Twelve percent of the school's population is Asian American, 8% is African American, 10% is Native American, and 70% is Caucasian. You are interested in being able to generalize the results of your survey to the entire student body. Describe a technique for obtaining a sample of size 100 that would have a reasonable chance of producing results that could be generalized to the entire school. Suppose you sent the surveys out to classes and asked the students to complete them and return them to the main office. Would the returns be likely to be biased in any direction? If so, why? In what direction? Suppose you decide to conduct the survey by personal interview. Members of the student council, all of whom happen to be Caucasian, will conduct personal interviews with the selected students. One of the survey questions reads, "Do you agree or disagree with the following statement: All students, regardless of ethnicity, have equal opportunity to become part of student government." What potential bias is present in this question?

Answers

Answer 1

Answers

We can use cluster sampling here. In this case, the population is divided into sampling units or clusters, for example, zone within educational or geographical areas. Having divided the population into clusters, we can use simple random sampling to select the clusters to be examined. Consider Asian American as one cluster, African american as another cluster and so on. According to the size of the cluster randomly select individuals from each cluster for a total of 100 individuals.

Generally there would be bias in accordance to the differnce in ethnicity. Also if the selected students are in a younger, then the teachers opnion would play a very big role there.

The question itself is a source of bias and it seems to give an idea to the students to think in these lines. The question can be better worded to get a correct answer. Even if there is equal opportunity, some students may read the question and give a negative answer.


Related Questions

Find T​, N​, and kappa for the plane curve Bold r left parenthesis t right parenthesis equalsleft parenthesis 7 Bold cos t plus 7 t Bold font size decreased by 1 sin t right parenthesis Bold i plus left parenthesis 7 Bold sin t minus 7 t Bold font size decreased by 1 cos t right parenthesis Bold j​, t greater than 0 .
Find T, N, and for the plane curve r(t) = (7 cost + 7t sin t)i + (7 sin t - 7t cos t)j, t> 0.

Answers

Answer:

Step-by-step explanation:

r(t) = (7 cost + 7t sin t)i + (7 sin t - 7t cos t)j

[tex]\frac{d \bar r t}{dt} =(7\frac{d}{dt}\cos t + 7\frac{d}{dt} (t \sin t)i+(7\frac{d}{dt} \sin t-7\frac{d}{dt} t \cos t)j[/tex]

[tex]=(7(-\sin t)+7(1* \sin t+t \cos t))i+(7 \cost -7(1*\cos t - t \sin t))j\\\\=7((-\sin t+\sin t+t \cos t)i+(\cos t-\cos t+t \sin t)j)\\\\=7((t\cos t)i+(t\sin t)j)[/tex]

[tex]\bar r'(t)=\frac{d \bar r t}{dt} =(7t\cos t)i+(7t\sin t)j---(1)\\\\11\bar r(t)=\sqrt{(7t\cos t)^2+(7t\sin t)^2}\\\\=\sqrt{49t^2(\cos^2t+\sin^2 t)} \\\\=7t[/tex]

[tex]\bar T (t)=\frac{\bar r'(t)}{11\bar r(t)11} =\frac{(7t\cos t)i+(7t\sin t)j}{7t} \\\\\barT(t)=(\cos t)i+(\sin t)j[/tex]

[tex]\bar T'(t)=\frac{d}{dt} (\cos t)i+\frac{d}{dt} (\sin t) j\\\\\bar T'(t)=(-\sin t)i+(\cos t)j---(2)\\\\11\bar T'(t)=\sqrt{(-\sin t)^2+(\cos t)^2} \\\\=\sqrt{\sin^2t+\cos^2t} \\\\=1[/tex]

[tex]\bar N(t)=\bar T'(t)=\frac{(-\sin t)i+(\cos t)j}{(1)} \\\\ \large \boxed {\bar N(t)=(-\sin t)i+(\cos t)j}[/tex]

[tex]K(t)=\frac{|\b\r T'(t)|}{\bar r (t)|} \\\\=\frac{|-\sin t i+\cos t j|}{|7t\cos t +7t \sin t j|}[/tex]

Using eq (1) and (2)

[tex]K(t)=\frac{\sqrt{(-\sin t)^2+(\cos t)^2} }{\sqrt{(7t\cos t)^2+(7t\sin t)^2} }\\\\=\frac{\sqrt{\sin^2 t+\cos^2t} }{\sqrt{49t^2(\cos^2 t+\sin^2t)} }\\\\=\frac{\sqrt{1} }{\sqrt{49t^2\times 1} } \\\\ \large \boxed {K(t)=\frac{1}{7t} }[/tex]

Apply the distributive property to create an equivalent expression.


1/2 (2a−6b+8) = ?

Answers

Answer:

a-3b+4

Step-by-step explanation:

1/2 (2a−6b+8) =

=1/2*2a-1/2*6b+1/2*8

=a-3b+4

Answer:

a - 3b + 4

Explanation:

According to the instructions, we must apply the distributive property to create an equivalent expression.

* reminder

distributive property formula: a (b + c) = ab + ac

Let's start by applying the distributive property to the expression.

[tex]\displaystyle\frac{1}{2} (2a - 6b + 8)\\\\\displaystyle\frac{1}{2} (2a) + \displaystyle\frac{1}{2} (-6b) + \displaystyle\frac{1}{2} (8)[/tex]

Simplify by multiplying.

[tex]\displaystyle\frac{1}{2}(2a)+\displaystyle\frac{1}{2}(-6b)+\displaystyle\frac{1}{2}(8)\\\\a+\displaystyle\frac{1}{2}(-6b)+\displaystyle\frac{1}{2}(8)\\\\a-3b+\displaystyle\frac{1}{2}(8)\\\\a-3b + 4[/tex]

Therefore, an equivalent expression to the given expression is a - 3b + 4.

POSSIBLE POINTS: 1
In the US, the average citizen receives an annual dose of 360 mrem of radiation. If eating a banana creates a 0.01 mrem dose, how many bananas
are equivalent to the annual dose of an average citizen?
720 bananas
36.000 bananas
25,000 bananas
180,000 bananas

Answers

Answer:

36,000

Step-by-step explanation:

So all we have to do is divide 360 by 0.01. that equals 36,000

the answer is 36,000 bananas

What type of probability is used in this scenario a bag has 10 red marbles 7 blue marbles and 12 yellow marbles the probability of drawing a yellow at random is 12/29

Answers

I could be wrong but i would want to say that's a ratio.

Answers? can someone help me please

Answers

i pretty sure the form is 2, and she bought 10 books, 5b-7=58
5b-7=58 hope I helped u

Find the value of the trig function indicated

Answers

Answer:

Step-by-step explanation:

base x height divided by 2

4x3=12

12 divided by 2 = 6

6x5= 30

Your answer would be sin 0= (3/5)
I got the because sin 0 = opposite/ hypotenuse

2.Sketch the region whose area is given by the integral and evaluate the integral.
3.Evaluate the given integral by changing to polar coordinates.....

Answers

2. The integration region,

[tex]\left\{(r,\theta)\mid\dfrac\pi6\le\theta\le\dfrac\pi2\land2\le r\le3\right\}[/tex]

corresponds to what you might call an "annular sector" (i.e. the analog of circular sector for the annulus or ring). In other words, it's the region between the two circles of radii [tex]r=2[/tex] and [tex]r=3[/tex], taken between the rays [tex]\theta=\frac\pi6[/tex] and [tex]\theta=\frac\pi2[/tex]. (The previous question of yours that I just posted an answer to has a similar region with slightly different parameters.)

You can separate the variables to compute the integral:

[tex]\displaystyle\int_{\pi/6}^{\pi/2}\int_2^3r^2\sin^2\theta\,\mathrm dr\,\mathrm d\theta=\left(\int_{\pi/6}^{\pi/2}\sin^2\theta\,\mathrm d\theta\right)\left(\int_2^3r^2\,\mathrm dr\right)[/tex]

which should be doable for you. You would find it has a value of 19/72*(3√3 + 4π).

3. Without knowing the definition of the region D, the best we can do is convert what we can to polar coordinates. Namely,

[tex]r^2=x^2+y^2[/tex]

so that

[tex]\displaystyle\iint_De^{x^2+y^2}\,\mathrm dA=\iint_Dre^{r^2}\,\mathrm dr\,\mathrm d\theta[/tex]

Someone let me know this answer

Answers

Hey baby I love you muahhh

WILL MARK BRAINLIEST

Sherlene bought 3 pounds of coffee and 2 pounds of chocolate for a total of $24. Which equation, written in standard form, correctly represents this scenario and correctly indicates what the
variables represent?

A) The correct equation is 3x + 2y = 24, where x is the price per pound of coffee and y is
the price per pound of chocolate.

B) The correct equation is 5x + y = 24, where x is the combined price per pound of
coffee and chocolate, and y is the total number of pounds purchased.

C) The correct equation is 3x + 2x + y = 24, where x is the combined price per pound of
coffee and chocolate, and y is the total number of pounds purchased.

D) The correct equation is 3x + 2y = 24, where x is the price per pound of chocolate and
y is the price per pound of coffee.

Answers

Answer:

AAAAAAAAAAAA answer is A

Step-by-step explanation:

The result of rounding the whole number 2,746,052 to the nearest hundred thousands place is:

Answers

Answer:

2700000

Step-by-step explanation:

Because it is not at 750000 it gets rounded down

Classify the triangle shown below. Check all that apply.
100°
40°
40°
A. Obtuse
B. Right
C. Isosceles
D. Equilateral
O E. Acute
F. Scalene​

Answers

Answer:

obtuse

isosceles

Step-by-step explanation:

It has one angle bigger than 90 so it is obtuse

It has two angles that measure the same so it has two sides that measure the same so it is isosceles

The given triangle is an Obtuse triangle.

What is mean by Triangle?

A triangle is a three sided polygon, which has three vertices and three angles which has the sum 180 degrees.

We have to given that;

All the angles are,

100°

40°

40°

Here, It has one angle bigger than 90 so it is obtuse.

And, It has two angles that measure the same so it has two sides that measure the same so it is isosceles.

Thus, The given triangle is an Obtuse triangle.

Learn more about the triangle visit;

brainly.com/question/1058720

#SPJ7

Solve for x in the following 4/2.6=5/x

Answers

The value of X is 3.25

Look at the attached picture

Hope it will help you

Good luck on your assignment

Use trigonometric substitution to evaluate the integral 13 + 12x − x2 dx . First, write the expression under the radical in an appropriate form so that a trigonometric substitution can be performed. 13 + 12x − x2

Answers

I don't see a square root sign anywhere, so I'll assume the integral is

[tex]\displaystyle\int\sqrt{13+12x-x^2}\,\mathrm dx[/tex]

First complete the square:

[tex]13+12x-x^2=49-(6-x)^2=7^2-(6-x)^2[/tex]

Now in the integral, substitute

[tex]6-x=7\sin t\implies\mathrm dx=-7\cos t\,\mathrm dt[/tex]

so that

[tex]t=\sin^{-1}\left(\dfrac{6-x}7\right)[/tex]

Under this change of variables, we have

[tex]7^2-(6-x)^2=7^2-7^2\sin^2t=7^2(1-\sin^2t)=7^2\cos^2t[/tex]

so that

[tex]\displaystyle\int\sqrt{13+12x-x^2}\,\mathrm dx=-7\int\sqrt{7^2\cos^2t}\,\cos t\,\mathrm dt=-49\int|\cos t|\cos t\,\mathrm dt[/tex]

Under the right conditions, namely that cos(t) > 0, we can further reduce the integrand to

[tex]|\cos t|\cos t=\cos^2t=\dfrac{1+\cos(2t)}2[/tex]

[tex]\displaystyle-49\int|\cos t|\cos t\,\mathrm dt=-\frac{49}2\int(1+\cos(2t))\,\mathrm dt=-\frac{49}2\left(t+\frac12\sin(2t)\right)+C[/tex]

Expand the sine term as

[tex]\dfrac12\sin(2t)}=\sin t\cos t[/tex]

Then

[tex]t=\sin^{-1}\left(\dfrac{6-x}7\right)\implies \sin t=\dfrac{6-x}7[/tex]

[tex]t=\sin^{-1}\left(\dfrac{6-x}7\right)\implies \cos t=\sqrt{7^2-(6-x)^2}=\sqrt{13+12x-x^2}[/tex]

So the integral is

[tex]\displaystyle-\frac{49}2\left(\sin^{-1}\left(\dfrac{6-x}7\right)+\dfrac{6-x}7\sqrt{13+12x-x^2}\right)+C[/tex]

What is the solution of StartRoot x minus 4 EndRoot + 5 = 2? x = –17 x = 13 x = 53 no solution

Answers

The solution to the given function is determined as 13.

Solution of the function

The solution of the function is calculated as follows;

[tex]\sqrt{x - 4} \ +\ 5 = 2[/tex]

collect similar terms together

 [tex]\sqrt{x - 4} \ = 2-5\\\\\sqrt{x - 4} \ = -3[/tex]

square both sides of the equation

[tex](\sqrt{x - 4} )^2 = (-3)^2\\\\x - 4 = 9\\\\x = 9 + 4\\\\x = 13[/tex]

Thus, the solution to the given function is determined as 13.

Learn more about solution of functions here: https://brainly.com/question/10439235

#SPJ9

Answer:

No solution

Step-by-step explanation:

Edge.

After four years in college, Josie owes $9500 in student loans. The interest rate on the federal loans is 11% and the rate on the private bank loans is 7%. The total interest she owed for one year was $901.00. What is the amount of each loan?

Answers

Step-by-step explanation:

federal= $4180

(9500*0.11*4)

private=$2660

(9500*0.07*4)

Between what two consecutive integers does the square root of 24 lie

Answers

4 and 5!

4 squared is 16, which is less than 24, and 5 squared is 25, which is more than 24!

[tex]\sqrt{24}[/tex] lies between two consecutive numbers  4  and 5

Given :

Given square root of 24

Lets write all the perfect square numbers

[tex]\sqrt{4}=2\\\sqrt{9}=3\\\sqrt{16} =4\\\sqrt{25} =5\\\sqrt{36} =6\\\sqrt{49}=7[/tex]

From the above perfect square root numbers, we can see that square root (24) lies between [tex]\sqrt{16} \; and\; \sqrt{25}[/tex]

So we can say that [tex]\sqrt{24}[/tex] lies between 4  and 5

Learn more : brainly.com/question/19028403

Graph a line with a slope of -5 that contains the
point (-3,-4).
y
6
.
.
2
-6
-4
2
4
-6

Answers

Answer:

Step-by-step explanation: idk

Ellie goes out to dinner with her family. The total bill is $57. They decide to leave a 20% tip. How much money should they leave as their tip?

Answers

Answer:

78doalrse

Step-by-step explanation:

solos w

Answer:

$11.40

Step-by-step explanation:

This question asks us to find how much money the family should leave as their tip.

To find the tip, multiply the cost of the total bill by the tip rate.

cost of bill * tip rate

The total bill was $57, and they are leaving a 20% tip

$57 * 20%

Convert 20% to a decimal by dividing by 100 or moving the decimal place 2 spaces to the left.

20/100=0.20

20.0--> 2.0 --> 0.20

20%=0.20

$57 * 0.20

Multiply 57 and 0.20

11.4= $11.40

The family should leave a tip of $11.40

Change2800cm squared into litres

Answers

Answer:

You cannot do this because 2800 cm squared is area and liters is volume. however, if you mean 2800 cm cubed, then the answer is 2.8 liters.

Step-by-step explanation:

√15(√6+6) A. √21+6√15 B. 3√10+6√15 C. √90+6 D. √90+6√15

Answers

Answer:

Before A. 32.7247330577, A. 27.8204757722, B. 32.7247330577, C. 15.4868329805, and D. 32.7247330577

Step-by-step explanation:

Used a calculator. Not that Hard.

Solve the equation sin sq x = 3cos sq x.

The value of x that satisfies the equation if x lies in the second quadrant is °.

The value of x that satisfies the equation if x lies in the third quadrant is

Answers

Answer:

Second quadrant = 120°.

Third quadrant = 210°

Step-by-step explanation:

We are given that:

[tex]sin^2(x) = 3cos^2(x)[/tex]

The following property is known:

[tex]sin^2(x) +cos^2(x)=1\\[/tex]

Combining both expressions:

[tex]sin^2(x) =1-cos^2(x)\\sin^2(x) = 3cos^2(x)\\\\1-cos^2(x) = 3cos^2(x)\\cos^2(x)=\frac{1}{4}\\cos(x)=\pm \frac{1}{2}[/tex]

If x lies in the second quadrant, then cos(x) = -1/2:

[tex]x=cos^{-1}(1/2)\\x=120^o[/tex]

The value of x that satisfies the equation if x lies in the second quadrant is 120°.

If x lies in the third quadrant, then cos(x) = -1/2:

[tex]x=120+90=210^o[/tex]

The value of x that satisfies the equation if x lies in the third quadrant is  210°

Answer:

The value of x that satisfies the equation if x lies in the second quadrant is 120

The value of x that satisfies the equation if x lies in the third quadrant is

240

Step-by-step explanation:

This is correct for Plato/Edmentum users :) Hope I could help !

Persia make a flower arrangement using the eight longest flowers which is the combined height of flowers Persia uses

Answers

Answer:

Persia should measure the length of each flower that she used in creating this flower arrangement and add up all the values to get the total height (i.e., the combined height) of the eight flowers used. Make sure to keep the units consistent all throughout the calculation to avoid any errors. For example, if centimetres are used to measure height of one flower, use centimetres all throughout and not switch to using inches at any point.

Hope that answers the question, have a great day!

Answer:

61 1/2

Step-by-step explanation:

math is ez

Consider the curve given by the equation (2y+1)^3 − 24x = −3.


(a) Show that dy/dx = 4/(2y+1)^2.

(b) Write an equation for the line tangent to the curve at the point (−1,−2).

(c) Evaluate d2y/dx2 at the point (−1,−2).

(d) The point (16,0) is on the curve. Find the value of (y−1)′(0).

Answers

Answer:

(a) dy/dx = 4/(2y+1)^2.

(b) y = 4/9 x - 14/9

(c) d2y/dx2 = -64/243

Step-by-step explanation:

You have the following equation

[tex](2y+1)^3-24x=-3[/tex]   (1)

(a) You first derivative implicitly the equation (1) respect to x:

[tex]\frac{d}{dx}[(2y+1)^3-24x]=\frac{d}{dx}[-3]\\\\3(2y+1)^2(2\frac{dy}{dx})-24=0[/tex]

next, you solve the last result for dy/dx:

[tex]6(2y+1)^2\frac{dy}{dx}=24\\\\\frac{dy}{dx}=\frac{4}{(2y+1)^2}[/tex](2)

(b) The equation for the tangent line is given by:

[tex]y-y_o=m(x-x_o)[/tex]    (3)

with yo = -2 and xo = -1

To find the slope m you use the result of the equation (2), because dy/dx evaluated in (-1,-2) is the slope at such point:

m = [tex]\frac{dy}{dx}=\frac{4}{(2(-2)+1)^2}=\frac{4}{9}[/tex]

Hence, by replacing in the equation (3) you obtain:

[tex]y-(-2)=\frac{4}{9}(x-(-1))\\\\y+2=\frac{4}{9}x+\frac{4}{9}\\\\y=\frac{4}{9}x-\frac{14}{9}[/tex]

hence, the equation for the tangent line is y = 4/9 x - 14/9

(c) To find d2y/dx2 you derivative the result obtain in the equation (2):

[tex]\frac{d^2y}{dx^2}=\frac{d}{dx}[4(2y+1)^{-2}]\\\\\frac{d^2y}{dx^2}=-8(2y+1)^{-3}(2\frac{dy}{dx})\\\\\frac{d^2y}{dx^2}=-16(2y+1)^{-3}\frac{dy}{dx}[/tex]     (4)

the second derivative for the point (-1,-2) is obtained by replacing y=-2 and dy/dx=m=4/9 in the equation (4):

[tex]\frac{d^2y}{dx^2}=-16(2(-2)+1)^{-3}(\frac{4}{9})=-\frac{64}{243}[/tex]

hence, d2y/dx2 evaluated in (-1,-2) is -64/243

Answer:

(A) The value of  [tex]dy/dx=\frac{4}{(2y+1)^2}[/tex].

(B) The equation of the tangent is : [tex]y=(4/9)x-(14/9)[/tex]

(C) The value of  [tex]\frac{d^2y}{dx^2}=-64/243[/tex]

(D) The point (16,0) is not on curve so it can not be determined by the given equation.

Step-by-step explanation:

Given information:

The equation [tex](2y+1)^3-24x=-3[/tex]

(A) For the first derivative of the given equation:

[tex]\frac{d}{dx}[(2y+1)^3-24x ]= \frac{d}{dx}(-3)[/tex]

[tex]3(2y+1)^2(dy/dx)-24=0[/tex]

[tex](dy/dx)=\frac{4}{(2y+1)^2}[/tex]

Hence , from the above equation it is shown that the value of

[tex]dy/dx=\frac{4}{(2y+1)^2}[/tex]

(B) The equation of the tangent to the curve is given by:

[tex]y-y_o=m(x-x_0)\\[/tex]

On putting the given values in the above equation

We get:

[tex]m=\frac{4}{(2(-2))+1)^2}[/tex]

[tex]m=4/9[/tex]

Hence, the equation of the tangent can be written as :

[tex]y-(-2)=(4/9)(x-(-1))\\y+2=\frac{4}{9}x+\frac{4}{9}[/tex]

So, the equation of the tangent is :

[tex]y=(4/9)x-(14/9)[/tex]

(C) Now ,

To find [tex]d^2y/dx^2[/tex] for the equation

We have to find double derivative of the equation;

[tex]\frac{d^2y}{dx^2} =\frac{d}{dx}[4(2y+1)^{-2}]\\\frac{d^2y}{dx^2} = -16(2y+1)^{-3}\frac{dy}{dx}[/tex]

On putting the values from the given information in the above equation;

[tex]\frac{d^2y}{dx^2} =-16(2(-2)+1)^{-3}(4/9)[/tex]

[tex]\frac{d^2y}{dx^2}=-64/243[/tex]

(D) For the equation [tex](2y+1)^3-24x=-3[/tex]

First check for the given points (16,0) if it satisfies the given equation or not.

Now on checking for the same the point is not satisfying the given equation hence, we can not find the value of [tex](y-1)'(0)[/tex].

For more information visit:

https://brainly.com/question/5797309?referrer=searchResults

Choose the function whose graph is given by:

Answers

The correct answer is b or y=sin(x-2) I think. Lol dumb trigonometry

The function whose graph is given is y = sin (x - 2).

Option B is the correct answer.

What is a function?

A function has an input and an output.

A function can be one-to-one or onto one.

It simply indicated the relationships between the input and the output.

Example:

f(x) = 2x + 1

f(1) = 2 + 1 = 3

f(2) = 2 x 2 + 1 = 4 + 1 = 5

The outputs of the functions are 3 and 5

The inputs of the function are 1 and 2.

We have,

The graph of y = sin(x - 2) is a sinusoidal function that is shifted 2 units to the right from the standard sine function y = sin(x).

The sine function oscillates between -1 and 1 as x increases, and the value of x at which the function reaches its minimum or maximum value is a multiple of π.

When we subtract 2 from x in the equation y = sin(x - 2), the entire graph is shifted to the right by 2 units, which means that the minimum and maximum points occur at x-values that are 2 units greater than they would be for the standard sine function.

The graph of y = sin(x - a) is a sinusoidal function that is shifted a units to the right from the standard sine function.

So, in this case, the graph of y = sin(x-2) looks like the standard sine function shifted 2 units to the right.

The amplitude and period of the function remain the same as the standard sine function, but the phase shift changes.

Thus,

The function whose graph is given is y = sin (x - 2).

Learn more about functions here:

https://brainly.com/question/28533782

#SPJ7

The highest and lowest scores on a test taken by 80 students are within 19 points of the average of the exam. The average for this exam was a 72. Set up an equation to solve for the highest and lowest scores. Show your original equation and the process used to find the highest and lowest exam score.

Answers

1. So start off by asking yourself, how will I get this answer? Absolute values!

2. So what the question is asking is for you to build and equation that shows us how to solve for the highest and lowest grades that are 19 points above and below the average of 72.

3. So your unknown is x, and for us to be able to plug in our highest and lowest scores it will always have to = 19 (that’s the rule we got from the question)

So now this is where your absolute value comes in.

4. Set it up

| x - 72 | = 19

5. Solve it

72-19=53 Minimum

72+19=91 Maximum

6. | 53 - 72 | = 19

| 91 - 72 | = 19

7x + 3 = 5 and y - 1= 6

Answers

Answer:

first answer is x= -5 second answer is y=7.

Step-by-step explanation:

7-5=2. 2+3=5.

7-1=6.

f f (x) = 5 x minus 25 and g (x) = one-fifth x + 5, which expression could be used to verify g(x) is the inverse of f(x)?

Answers

Answer:

We study if the composition of both functions equals the identity ("x"), that is if

[tex]f(g(x))=x[/tex]

Step-by-step explanation:

The composition of the two functions should render 'x" if one is the inverse of the other. That is, we need to find what  [tex]f(g(x))[/tex] renders. Notice as well that the same verification could be done with examining  [tex]g(f(x))[/tex].

Let's work with [tex]f(g(x))[/tex] :

[tex]f(g(x))=f(\frac{1}{5} x+5)=5\,(\frac{1}{5} x+5)-25= x+25-25=x[/tex]

So we see that the composition of both functions indeed render "x", and that way we have verified that one is the the inverse of the other.

Answer:

B. One-fifth (5 x minus 25) + 5

Step-by-step explanation:

Just got it right on the test.

Luke collected 1,034 baseball cards, 1,289 football cards, and 1,566 hockey cards. Use mental math to find the number of cards in Luke’s collection. Solve this problem any way you choose.

Answers

Answer:

3889

Step-by-step explanation:

you add all the numbers to get the answer

Answer:3889

Step-by-step explanation:So what u want to do is add them

1034

1289

1566

3889

Suppose a large shipment of stereos contained 18% defectives. If a sample of size 306 is selected, what is the probability that the sample proportion will differ from the population proportion by less than 6%

Answers

Answer:

99.36% probability that the sample proportion will differ from the population proportion by less than 6%

Step-by-step explanation:

To solve this question, we need to understand the normal probability distribution and the central limit theorem.

Normal probability distribution

Problems of normally distributed samples are solved using the z-score formula.

In a set with mean [tex]\mu[/tex] and standard deviation [tex]\sigma[/tex], the zscore of a measure X is given by:

[tex]Z = \frac{X - \mu}{\sigma}[/tex]

The Z-score measures how many standard deviations the measure is from the mean. After finding the Z-score, we look at the z-score table and find the p-value associated with this z-score. This p-value is the probability that the value of the measure is smaller than X, that is, the percentile of X. Subtracting 1 by the pvalue, we get the probability that the value of the measure is greater than X.

Central Limit Theorem

The Central Limit Theorem estabilishes that, for a normally distributed random variable X, with mean [tex]\mu[/tex] and standard deviation [tex]\sigma[/tex], the sampling distribution of the sample means with size n can be approximated to a normal distribution with mean [tex]\mu[/tex] and standard deviation [tex]s = \frac{\sigma}{\sqrt{n}}[/tex].

For a skewed variable, the Central Limit Theorem can also be applied, as long as n is at least 30.

For a sample proportion p in a sample of size n, we have that the sampling distribution of the sample proportions has [tex]\mu = p, s = \sqrt{\frac{p(1-p)}{n}}[/tex].

In this question:

[tex]n = 306, p = 0.18, \mu = 0.18, s = \sqrt{\frac{0.18*0.82}{306}} = 0.0220[/tex].

What is the probability that the sample proportion will differ from the population proportion by less than 6%

This is the pvalue of Z when X = 0.18 + 0.06 = 0.24 subtracted by the pvalue of Z when X = 0.18 - 0.06 = 0.12. So

X = 0.24

[tex]Z = \frac{X - \mu}{\sigma}[/tex]

By the Central Limit Theorem

[tex]Z = \frac{X - \mu}{\sigma}[/tex]

[tex]Z = \frac{0.24 - 0.18}{0.022}[/tex]

[tex]Z = 2.73[/tex]

[tex]Z = 2.73[/tex] has a pvalue of 0.9968

X = 0.12

[tex]Z = \frac{X - \mu}{\sigma}[/tex]

[tex]Z = \frac{0.12 - 0.18}{0.022}[/tex]

[tex]Z = -2.73[/tex]

[tex]Z = -2.73[/tex] has a pvalue of 0.0032

0.9968 - 0.0032 = 0.9936

99.36% probability that the sample proportion will differ from the population proportion by less than 6%

You have received an order of 100 robotic resistance spot welders which contains 5 defective welders. You randomly select 15 welders from the order without replacement to inspect to check whether they are defective.
(a) Determine the PMF of the number of defective welders in your sample? Remember to list all possible values of the random variable.
(b) Determine the probability that there are at least 4 defective welders in the sample? Hint: No need to calculate the final numerical results. Appropriately plugging in numbers in the mathematical expression is sufficient

Answers

Answer:

a)

[tex]P(X = 0) = h(0,100,15,5) = \frac{C_{5,0}*C_{95,15}}{C_{100,15}} = 0.4357[/tex]

[tex]P(X = 1) = h(1,100,15,5) = \frac{C_{5,1}*C_{95,14}}{C_{100,15}} = 0.4034[/tex]

[tex]P(X = 2) = h(2,100,15,5) = \frac{C_{5,2}*C_{95,13}}{C_{100,15}} = 0.1377[/tex]

[tex]P(X = 3) = h(3,100,15,5) = \frac{C_{5,3}*C_{95,12}}{C_{100,15}} = 0.0216[/tex]

[tex]P(X = 4) = h(4,100,15,5) = \frac{C_{5,4}*C_{95,11}}{C_{100,15}} = 0.0015[/tex]

[tex]P(X = 5) = h(5,100,15,5) = \frac{C_{5,5}*C_{95,10}}{C_{100,15}} = 0.00004[/tex]

b) 0.154% probability that there are at least 4 defective welders in the sample

Step-by-step explanation:

The welders are chosen without replacement, so the hypergeometric distribution is used.

The probability of x sucesses is given by the following formula:

[tex]P(X = x) = h(x,N,n,k) = \frac{C_{k,x}*C_{N-k,n-x}}{C_{N,n}}[/tex]

In which

x is the number of sucesses.

N is the size of the population.

n is the size of the sample.

k is the total number of desired outcomes.

Combinations formula:

[tex]C_{n,x}[/tex] is the number of different combinations of x objects from a set of n elements, given by the following formula.

[tex]C_{n,x} = \frac{n!}{x!(n-x)!}[/tex]

In this question:

100 welders, so [tex]N = 100[/tex]

Sample of 15, so [tex]n = 15[/tex]

In total, 5 defective, so [tex]k = 5[/tex]

(a) Determine the PMF of the number of defective welders in your sample?

There are 5 defective, so this is P(X = 0) to P(X = 5). Then

[tex]P(X = x) = h(x,N,n,k) = \frac{C_{k,x}*C_{N-k,n-x}}{C_{N,n}}[/tex]

[tex]P(X = 0) = h(0,100,15,5) = \frac{C_{5,0}*C_{95,15}}{C_{100,15}} = 0.4357[/tex]

[tex]P(X = 1) = h(1,100,15,5) = \frac{C_{5,1}*C_{95,14}}{C_{100,15}} = 0.4034[/tex]

[tex]P(X = 2) = h(2,100,15,5) = \frac{C_{5,2}*C_{95,13}}{C_{100,15}} = 0.1377[/tex]

[tex]P(X = 3) = h(3,100,15,5) = \frac{C_{5,3}*C_{95,12}}{C_{100,15}} = 0.0216[/tex]

[tex]P(X = 4) = h(4,100,15,5) = \frac{C_{5,4}*C_{95,11}}{C_{100,15}} = 0.0015[/tex]

[tex]P(X = 5) = h(5,100,15,5) = \frac{C_{5,5}*C_{95,10}}{C_{100,15}} = 0.00004[/tex]

(b) Determine the probability that there are at least 4 defective welders in the sample?

[tex]P(X \geq 4) = P(X = 4) + P(X = 5) = 0.0015 + 0.00004 = 0.00154[/tex]

0.154% probability that there are at least 4 defective welders in the sample

Other Questions
triangle ABC a = 4 and m angle C=30 If the area of the triangle is 6 what is the length of side b PLZ HELP! WILL GIVE BRAINIEST!! Please help me understand this!! 0.76 g of lead(2) nitrate was dissolvedin 50.00ml of water a d treated with 25.00 ml of 0.2010M sodium sulfate inoder to determine a content of lead 2 ion. A white precipitate was formed and it was collected and dried. Calculate the amount of precipitate formed in this reaction Easy points for yall mathy people, fairly easy question. Its in the photo!! Please answer correctly !!!!!!!!! Will mark brianliest !!!!!!!!!!!! 4. Fill in the blanks with the words from the Word Bank below. How did Prince Henry the Navigator and Amerigo Vespucci inspire the spirit of adventure? Please answer A concrete driveway is 5 feet longer than twice its width. It has a brick walkway 3 feet wide on the front and side of the driveway. The area of the brick walkway is 213square feet. Find the dimensions of the driveway. An insurer wishes to compare the information given in an insurance application with previous insurance applications by the same applicant but for different companies. What organization can help the insurer accomplish this? Carole ordered 4 dresses for $80 each, a $25 sweater, and a coat. The cost of the items without sales tax was $430. What was the cost of the coat? The war in lraq cost the United States about? which letter best shows the location of 9/7?A: AB: BC: CD: Dill give brainliest and 10 points answer fasttttt Originally the dimensions of a rectangle were 20cm by 23cm. When both dimensions were decreased by the same amount, the area of the rectangle decreases by 120. Find the dimensions of the rectangle. Will give BRAINLIEST How did the universe begin? The BLANK is an international organization that does research and creates policy on disease prevention, including BLANK strategies. First person to answer right gets Brainliest and 5 points! help me help me help me asap You deposit $2000 in a savings account at Hometown Bank, which has a rate of 6%.Find the amount, A, of money in the account after 3 years subject to interestcompounded once a year.b) Find the interest. Find the product. (3x 2 - 5x + 3)(3x - 2) If 0.092J of heat causes a 0.267 degree C temperature change, what mass of water is present?